subject
Mathematics, 20.03.2021 22:20 leo4687

What is the correct answer?


What is the correct answer?

ansver
Answers: 2

Another question on Mathematics

question
Mathematics, 21.06.2019 17:00
The magnitude, m, of an earthquake is defined to be m=log l/s, where i is the intensity of the earthquake (measured by the amplitude of the seismograph wave) and s is the intensity of a “standard” earthquake, which is barely detectable. what is the magnitude of an earthquake that is 1,000 times more intense than a standard earthquake? use a calculator. round your answer to the nearest tenth.
Answers: 1
question
Mathematics, 21.06.2019 23:10
Click an item in the list or group of pictures at the bottom of the problem and, holding the button down, drag it into the correct position in the answer box. release your mouse button when the item is place. if you change your mind, drag the item to the trashcan. click the trashcan to clear all your answers. find the lateral area for the prism. l.a. = 0123456789
Answers: 3
question
Mathematics, 21.06.2019 23:30
Jane has 5 nickels.john has 3 dimes.jane says she has a greater amount of money.with the information given,is she correct?
Answers: 3
question
Mathematics, 22.06.2019 00:30
I've been working on this for a few days and i just don't understand, it's due in a few hours. you. the direction of a vector is defined as the angle of the vector in relation to a horizontal line. as a standard, this angle is measured counterclockwise from the positive x-axis. the direction or angle of v in the diagram is α. part a: how can you use trigonometric ratios to calculate the direction α of a general vector v = < x, y> similar to the diagram? part b suppose that vector v lies in quadrant ii, quadrant iii, or quadrant iv. how can you use trigonometric ratios to calculate the direction (i.e., angle) of the vector in each of these quadrants with respect to the positive x-axis? the angle between the vector and the positive x-axis will be greater than 90 degrees in each case. part c now try a numerical problem. what is the direction of the vector w = < -1, 6 > ?
Answers: 1
You know the right answer?
What is the correct answer?
...
Questions
question
Mathematics, 13.10.2020 14:01
question
Mathematics, 13.10.2020 14:01
question
Biology, 13.10.2020 14:01
Questions on the website: 13722367